Вы находитесь на странице: 1из 11

PROBLEMAS SOBRE PONTOS I

Davi Maximo (UFC) e Samuel Feitosa (UFC)


Distribuir pontos num plano ou num espao uma tarefa que pode ser realizada de
forma muito arbitrria. Por isso, problemas sobre pontos podem ser de diversas naturezas. Nesse
artigo, trataremos as principais tcnicas para resolver esses tipos de problemas,

1. Fecho Convexo
Pense no seguinte: dados n pontos num plano, podemos escolher alguns deles formando
um nico polgono convexo que contm, junto com seu bordo e seu interior, todos os n pontos.
Tal afirmao pode ser provada por induo (que alias, uma ferramenta que sempre deve ser
lembrada em problemas de matemtica discreta em geral). Tal polgono chamado o fecho
convexo desses n pontos. Vamos ver que to pouco j nos ajuda bastante em alguns problemas
sobre pontos.
PROBLEMA 1

Seja S um conjunto finito de pontos, no havendo trs colineares, tal que dados quaisquer 4
pontos de S eles formam um quadriltero convexo. Mostre que S um conjunto de vrtices de
um polgono convexo.
SOLUO

Seja H o fecho convexo de S.

.
P
H

B
A

Suponha um ponto P de S no interior de H.


Escolha uma triangulao de H (assim como
o fecho convexo, simples provar que todo
polgono convexo admite pode ser dividido
por tringulos tendo como lados diagonais ou
lados do polgono, tente induo).
Assim, P fica no interior de algum tringulo
ABC. Logo, o quadriltero ABCP no
convexo, absurdo!
Portanto, S no pode ter pontos no interior do
seu fecho convexo, donde S convexo, j
que S no contm trs pontos colineares.

Os prximos problemas so resolvidos similarmente.


PROBLEMA 2

Mostre que dados 5 pontos , no trs colineares, existe um quadriltero convexo com vrtices
nesses pontos.
PROBLEMA 3

Mostre que dado qualquer conjunto de pontos no plano existe uma reta por dois destes pontos
que divide o plano em dois semi-planos de modo que um desses semi-planos no contm
nenhum ponto do conjunto.
PROBLEMA 4

(Lista Cone Sul 2001) possvel que a reunio de um nmero finito de quadrilteros no
convexos seja um polgono convexo?

PROBLEMA 5

Dado um conjunto de N discos de raios unitrios. Esses crculos podem se intersectar (mas no
coincidir). Mostre que existe um arco de comprimento maior ou igual a

2
pertencendo
N

circunferncia de um desses discos que no coberto por nenhum outro disco.


(IDIA DA SOLUO) Consideremos o fecho

convexo H desse conjunto de discos. Um arco


que esteja na borda do fecho convexo no pode
ser coberto por outro disco. Mostre que a
juno de todos os arco no bordo de H um
crculo de raio unitrio. Como este crculo tem
permetro 2 e no mximo juntamos N arcos,
pelo menos um dos arcos da juno maior ou
2
igual a
.
N
PROBLEMA 6

(OBM 96) Existe um conjunto A de n pontos ( n 3 ) em um plano tal que:


i) A no contm trs pontos colineares;
ii) dados quaisquer trs pontos pertencentes a A, o centro da circunferncia que contm estes
pontos tambm pertence a A?
Os prximos dois problemas so de IMO e podem ser resolvidos usando s fecho
convexo (na realidade, muita raa tambm, que algo imprescindvel em qualquer problema,
principalmente de IMO).
PROBLEMA 7

(IMO 99/1) Determine todos os conjuntos finitos S de pontos do plano com pelo menos trs
elementos que satisfazem a seguinte condio:
Para quaisquer dois pontos distintos A e B de S, a mediatriz do segmento AB um eixo de
simetria de S.
(Veja a soluo desse problema por Fabrcio Siqueira Benevides na Revista Eureka! N6. )
PROBLEMA 8

3 para os quais existem n pontos A1 , A2 ,

(IMO 95/3) Determine todos os inteiros n


plano, e nmeros reais r1 , r2 ,

, An no

rn satisfazendo as condies:

(a) no h trs pontos Ai , A j , Ak colineares;


(b) para cada tripla i ,j, k ( 1

n ) o tringulo Ai A j Ak tem rea ri

rj

rk .

SOLUO

Vamos fazer o caso n 5 . Considere, dentre os n pontos, cinco pontos A1 , A2 , A3 , A4 , A5 e


seu fecho convexo . Temos trs casos:
1 Caso: O Fecho Convexo um tringulo.
Podemos assumir que tal fecho o tringulo A1 A2 A3 , A4 e A5 esto no interior de A1 A2 A3 ,
com A5 fora de A1 A2 A4 e A4 fora de A1 A2 A4 (faa uma figura). Seguindo nossa notao para
reas, temos:

[ A1 A4 A5 ] [ A1 A2 A3 ]
(r1

r2

r4 ) (r1

(r1
r3

r4

r5 ) (r1

r2

r3 )

r5 ) [ A1 A2 A4 ] [ A1 A3 A5 ] [ A1 A2 A3 ].

absurdo!
2 Caso: O Fecho Convexo um quadriltero
Suponha A5 no interior do fecho convexo A1 A2 A3 A4 . Note que

[ A1 A2 A3 A4 ] [ A1 A2 A3 ] [ A1 A3 A4 ] [ A1 A2 A4 ] [ A2 A3 A4 ]
e portanto, (r1 r2 r3 ) ( r3 r4 r1 ) (r1 r2 r4 ) ( r2 r4 r3 )
r1 r3
Logo, 2[ A1 A2 A3 A4 ] 3(r1 r2 r3 r4 ) . Tambm,
[ A1 A2 A3 A4 ] [ A1 A2 A5 ] [ A2 A3 A5 ] [ A3 A4 A5 ] [ A4 A1 A5 ]
Logo, temos r5

(r1

r2

r3

r4 ) 8

r2

r4 .

[ A1 A2 A3 A4 ] 12

0 . Agora, observe que como


A1 , A3 , A5 no so colineares, podemos supor um dos lados de A1 A3 A5 180 . Ento, um
dos quadrilteros A1 A5 A3 A4 , A1 A5 A3 A2 convexo. Digamos, A1 A5 A3 A4 convexo. Ento,
temos r1 r3 r4 r5 e portanto, ficamos com r2 r5 . Analogamente, usando que A2 , A4 , A5
no so colineares, temos r5 r1 ou r3 . Assim, trs dos nmeros r1 , r2 , r3, r4 , r5 so negativos,
obtendo uma rea negativa. Absurdo!
3 Caso: O Fecho convexo um pentgono
Suponha que r1 seja o menor deles. Traando uma paralela l por A1 reta A3 A4 . Como

[ A1 A3 A4 ]

r1

r3

r4

r2

r3

r4

[ A2 A3 A4 ] , A2 pertence a l ou ao semiplano definido

por l oposto ao A3 A4 e, analogamente A5 . Como A1 , A2 , A5 no podem estar todos em l ,


temos

A2 A1 A5

Logo, n

r1

r2

180 , absurdo! .

4 . Um exemplo para n

r3

r4

4 um quadrado A1 A2 A3 A4 de lado 1 com

1 6.

Finalizamos essa parte com dois problemas bonitinhos.


PROBLEMA 9

(USAMO 2005) Seja n um inteiro positivo maior que 1. Suponha que so dados 2n pontos no
plano, no havendo trs colineares. Suponha que n dos 2n so pintados de azul e os outros n de
vermelho. Uma reta no plano dita balanceada se passa por um ponto azul e um ponto
vermelho, e o nmero de pontos azuis em cada um de seus lados igual ao nmero de pontos
vermelhos. Prove que existem pelo menos duas retas balanceadas.
DICA: Prove que cada ponto do fecho convexo dos pontos est em pelo menos uma reta

balanceada.
PROBLEMA 10

(Kmal 2002) Dado um conjunto qualquer de pontos no plano, no contendo trs colineares,
prove que possvel colorir os pontos com duas cores (azul e vermelho) tal que todo semiplano
contendo pelo menos trs pontos do conjunto contenha um ponto de cada cor.

2. Princpio das Casas dos Pombos


O princpio da casas dos Pombos, PCP, importante e deve ser lembrado sempre.
Nossos dois primeiros problemas dessa sesso so apenas verses dificultadas daquele exerccio
clssico do PCP. : dados cincos pontos num quadrado unitrio, existem dois cuja distncia entre
eles menor que

PROBLEMA 11

(Japo 97) Prove que entre quaisquer dez pontos no interior de um crculo de dimetro 5,
existem dois cuja distncia entre eles menor que 2.
PROBLEMA 12

(Coria 97) Prove que entre quaisquer quatro pontos no interior de um crculo unitrio, existem
dois deles cuja distncia menor que 2 .
PROBLEMA 13

(Rioplatense 2002) Daniel escolhe um inteiro positivo n e diz a Ana. Com esta informao, Ana
escolhe um inteiro k e diz a Daniel. Daniel traa ento n circunferncias em um papel e escolhe
k pontos distintos com a condio de que cada um deles pertena a alguma das circunferncias
que traou. Em seguida, apaga as circunferncias que traou, sobrando visveis apenas os k
pontos que marcou. A partir desses pontos, Ana deve reconstruir pelo menos uma das
circunferncias que traou Daniel. Determinar qual o menor valor de k que permite Ana
alcanar seu objetivo independente como Daniel escolha as n circunferncias e os k pontos.
SOLUO

O valor mnino k 2n 2 1 .
1 Passo: k 2n 2 1 suficiente.
Se so dados 2n 2 1 pontos marcados por Daniel, como estes pontos so distribudos em n
circunferncias, pelo Princpio das Casas dos Pombos, pelo menos 2n+1 deles esto em uma
mesma circunferncia traada por Daniel. Ento, se Ana traa todas as circunferncias
determinadas por estes 2n 2 1 , haver uma delas, digamos , com pelo menos 2n+1 dos
pontos. Como estes 2n+1 pontos provm das n circunferncias de Daniel, trs deles esto numa
mesma circunferncia traada por ele, digamos
. Logo,
e
tm pelo menos trs pontos
em comum, e portanto, so a mesma circunferncia (abusando da notao:
). Assim, Ana
consegue determinar umas das circunferncias traadas por Daniel.
2 Passo: Se k 2n 2 1 , Daniel pode traar circunferncias e escolher k pontos de modo a
tornar impossvel para Ana determinar tais circunferncias.
Basta considerar k 2n 2 :

Traamos n circunferncias concntricas


, n e outras n circunferncias
1, 2 ,
, n duas a duas disjuntas, de
1, 2 ,

modo que

2
1

corta

em dois pontos

distintos, para i, j= 1,2,...,n. H exatamente


2n 2 pontos de interseco. Se Daniel
marca estes pontos e apagas suas
circunferncias, Ana no conseguir
reconstruir com certeza nenhuma das
circunferncias, pois Daniel pode ter
traado inicialmente tanto 1 , 2 , , n
quanto 1 , 2 , , n .

PROBLEMA 14

(Rioplatense 1999) Dois jogadores A e B disputam o seguinte jogo: A escolhe um ponto de


coordenadas inteiras do plano e o pinta de verde; em seguida B escolhe 10 pontos de
coordenadas inteiras, ainda no coloridos e os pinta de amarelo. O jogo continua assim com as
mesmas regras: A e B escolhem um e dez pontos ainda no coloridos e os pintam de verde e
amarelo, respectivamente.
(a) O objetivo de A obter 1112 pontos verdes que sejam as intersees de 111 retas
horizontais e 111 retas verticais (i.e., paralelas aos eixos de coordenadas). O objetivo de B
impedir-lhe. Determine qual dos jogadores tem uma estratgia vencedora que lhe assegura seu
objetivo.
(b) O objetivo de A obter quatro pontos verdes que sejam vrtices de um quadrado de lados
paralelos aos eixos coordenados. O objetivo de B impedir-lhe. Determine qual dos jogadores
tem uma estratgia vencedora que lhe assegura seu objetivo.

3. Idias Extremais
Na matemtica em geral, problemas de existncia so muito comuns e importantes. So
aqueles problemas que nos pedem para provar que a existncia de alguma coisa. Na seo
anterior, no explicitamente, nos deparamos com problemas desse tipo. no foi para vender o
artigo que iniciamos ambas as sees dizendo falando da importncia dessas idias, mas pelo o
fato de que o PCP e o Princpio Extremal juntos so as ferramentas mais indispensveis para o
ataque desses problemas.
Mas afinal, que Princpio Extremal esse? Digamos que temos um problema onde nos
pedido para provar a existncia de um elemento satisfazendo uma certa propriedade P. Ento,
ns escolhemos um elemento que satisfaz maximalmente ou minimalmente, ou seja,
extremalmente (ser que acabamos de inventar essas palavras?) uma outra propriedade Q, no
acidentalmente ligada com a desejada propriedade P. O que ser que isso nos d? Vejamos
alguns problemas.
PROBLEMA 15

(Austrlia 91) So dados n 3 pontos no plano tais que a rea de um tringulo formado por
quaisquer trs deles no mximo 1. Prove que os n pontos esto em um tringulo de rea no
mximo 4.
SOLUO

Sejam P1 , P2 , , Pn os n pontos. Dentre os tringulos considerados, seja ABC o de maior rea


(o cara com a propriedade Q). Considere por A uma reta a paralela a BC. Sendo assim, qualquer
outro ponto Pi

PZ
B

deve estar no mesmo semiplano de B e C definido por


A, pois do contrrio teramos um absurdo
[ PBC ] [ ABC ] (aqui [X], denota a rea de X) .
Analogamente, considerando as retas b e c por B e C
paralelas a AC e AB, respectivamente, conclumos que
todos os pontos P devem estar no tringulo XYZ
(acompanhe a figura ao lado). Como,
[ XYZ ] 4[ ABC ] 4 1 4 , o resultado segue. (isto ,
XYZ satisfaz a propriedade P).

PROBLEMA 16

(Putnam 1979) Sejam 2n pontos no plano escolhidos de modo que quaisquer 3 no so


colineares, n deles so pintados de vermelho e n deles so pintados de azul. Prove que possvel

parear os pontos usando segmentos ligando cada ponto vermelho a exatamente um ponto azul de
modo que esses segmentos no se cortem.
SOLUO

Existem n 2 maneiras de parear esses pontos. claro que alguns desses pareamentos no
cumprem a condio do enunciado. Olhemos em cada pareamento a soma dos seus segmentos.
Escolha o pareamento que tem soma mnima. Suponha que nele existem dois segmentos AB e
CD que se cortam (com A e C vermelhos)

Pela desigualdade triangular temos:


AO OD CD
AB CD AD CB
OB OC CB
Logo se trocarmos AB e CD por AD e CB
diminuiremos nossa soma. Assim neste pareamento
no temos dois segmentos que se cortam.

PROBLEMA 17

(Teorema de Sylvester) Um conjunto S de pontos no plano tem a seguinte propriedade: qualquer


reta passando por 2 pontos passa tambm por um terceiro. Mostre que todos os pontos esto
sobre uma reta.
SOLUO

Considere o conjunto L das retas que passam por dois pontos de S. Cada ponto de S tem uma
distncia associada a cada reta de L. Como L e S so conjuntos finitos ento temos um nmero
finito distncias. Considere o par (l , s ) do ponto s S e l L com a menor distncia no
nula associada. Como l passa por dois pontos de S ento dever passar por um terceiro. Pelo
menos dois pontos de S , digamos A e B, devero estar em um mesmo lado de l determinado
por P (p da perpendicular de s at l ).

Suponhamos que A esteja entre B e P. Seja m a reta


que passa por B e s ento:
distncia( A, m) distncia( P, m) distncia( s, l )
Absurdo!
Assim todas as distncias associadas tm que ser zero! Todos os pontos so colineares!
A seguir, veja como usar Sylvester.
PROBLEMA 18

So dados (N 3) pontos no plano, nem todos colineares. Mostre que so necessrios pelo
menos n retas para unir todos os possveis pares de pontos.
SOLUO

Vamos tentar usar induo. Se N 3 os trs pontos formaro um tringulo. As retas suportes
dos trs lados desse tringulo satisfazem nossa afirmao. Suponha que a afirmao seja vlida
para N k . Considere um conjunto T de N k 1 pontos. Como nem todos esses pontos
esto sobre uma mesma reta decorre do teorema de Sylvester que existe uma reta que passa por
apenas dois pontos (A e B) do conjunto. Pelo menos um dos conjuntos T \ { A} ou T \ {B} no
poder ter todos os seus k pontos colineares. Ento pela hiptese teremos pelo menos k retas,
mas a reta AB no foi contada, assim a afirmao tambm verdadeira para N k 1 .

PROBLEMA 19

(Ibero 93) Prove que para qualquer polgono convexo de rea 1, existe um paralelogramo de
rea dois que o contm.
PROBLEMA 20

(OBM 94) Considere todos os crculos cujas circunferncias passam por trs vrtices
consecutivos de um polgono convexo. Prove que um destes crculos contm todo o polgono.
PROBLEMA 21

(Rioplatense 97) Agustina e Santiago jogam o seguinte jogo sobre uma loja retangular:
Agustina diz um nmero n. Santiago, ento marca n pontos sobre a loja. Em seguida, Agustina
escolhe alguns dos pontos marcados por Santiago. Santiago ganha o jogo se consegue desenhar
um retngulo com lados paralelos aos da loja, que contenha todos os escolhidos por Agustina e
nenhum dos restantes. Do contrrio, Agustina ganha.
Qual o menor nmero que deve escolher Agustina para assegurar-se da vitria, independente
como jogue Santiago?
PROBLEMA 22

(Rssia 2000) So dados 2n+1 segmentos em uma linha reta. Cada segmento intersecta pelo
menos n outros. Prove que um desses segmentos intersecta todos os outros.
PROBLEMA 23

(Japo 2002) dado um conjunto S de 2002 pontos no plano xy, no havendo dois deles com a
mesma abscissa x ou ordenada y. Para quaisquer dois desses pontos P e Q , considere o
retngulo cuja diagonal PQ e cujos os lados so paralelos aos eixos. Denotemos por WPQ o
nmero de pontos de S no interior desse retngulo, sem contar com P e Q. Determine o maior
valor N possvel que satisfaz: no importa como os pontos de S esto arranjados, existe pelo
menos um par P e Q deles com WPQ N .
PROBLEMA 24

Dados 2n+2 pontos no plano, no havendo trs colineares, prove que existem dois deles que
determinam uma reta que dos 2n pontos restantes, separa n deles dos outros n.
PROBLEMA 25

(Banco IMO 93) Dados 2n+3 pontos num plano, no havendo trs colineares nem quatro
concclicos, prove que podemos escolher trs deles de modo o crculo passando por estes tem n
dos pontos restantes no seu interior e n no exterior.
SOLUO

A2n

P
Q

Atravs do fecho convexo dos pontos,


escolha dois deles, digamos P e Q, tais que
todos os outros pontos esto no mesmo
semi-plano definido pela reta PQ.
Os outros 2n+1 pontos podemos ordenar
A1 , A2 , , A2 n 1 de modo que:

PA1Q
A1

An

PA2 Q

PA2 n 1Q ( ) ,

onde as desigualdades so todas


restritas, j que no h 4 pontos
concclicos.
Considerando os crculos por P, Ai , Q ,
i 1,2, ,2n 1 , vemos que o crculo
por P, An 1 , Q satisfaz o pedido.

PROBLEMA 26

So dados n pontos num plano. Em cada ponto mdio de um segmento ligando dois desses
pontos, colocamos um marcador. Prove que pelo menos 2n 3 marcadores so utilizados.

4. Problemas de Coberturas
Nos problemas sobre pontos at agora, ficou claro que um pouco de geometria
(sinttica, analtica, trigonomtrica ou utilizando o plano complexo) pode ser til. Finalizaremos
esse artigo com uma seo falando um pouco disso, em particular, fazendo coberturas com
crculos.
PROBLEMA 27

Seja C um crculo de raio 16 e A um anel tendo raio interno 2 e raio externo 3. Agora suponha
que um conjunto S de 650 pontos selecionado dentro de C. Prove que, no importa como os
pontos de S so selecionados dentro de C, o anel A pode ser colocado de modo a cobrir pelo
menos 10 pontos de S.
SOLUO

Queremos mostrar que existe um ponto X no plano que possui uma distncia maior que 2 e
menor que 3 pelo menos 10 pontos de S. Sobre cada ponto de S coloque um anel A. Basta
mostrarmos que existe um ponto X que est no interior de pelo menos 10 desses anis. As
intersees desses anis produzem pequenas regies (veja a figura).

Veja que existem trs pequenas regies que esto em


dois anis e uma que est em trs. Somando as reas dos
trs anis contaremos trs regies duas vezes e uma trs
vezes. Somando a rea de cada anel temos
650.(9
4 ) 3250 .Aumentando o raio do crculo C
para 19 poderemos cobrir todos esses anis. Se cada
pequena regio foi contada no mximo 9 vezes
contaremos no mximo 9 vezes a rea desse novo
crculo , ou seja, 9.19 2
3249
3250 .
Assim existir uma pequena regio contida em pelo menos 10 anis. Basta escolhermos um
ponto X dessa regio.
PROBLEMA 28

(Ibero 98) Encontre o maior inteiro n para o qual existem pontos P1 , P2 ,..., Pn no plano e
nmeros reais r1 , r2 ,..., rn tais que a distncia entre Pi e Pj ri

rj .

PROBLEMA 29

(Teste de Seleo da Romnia para a IMO-1978) M um conjunto de 3n pontos no plano tal


que a maior distncia entre quaisquer dois desses pontos 1 unidade. Prove que:
a. Para quaisquer 4 pontos de M, a distncia entre algum par de pontos pelo menos

3
cobre todo o conjunto M.
2
Existe algum par entre os 3n pontos de M cuja distncia entre eles no mximo
4

b. Algum crculo de raio


c.

(3 n

3)

SOLUO

a. Vamos tentar arranjar um tringulo no acutngulo em M. Considere o fecho convexo de


quatro pontos de M. Podemos ter um quadriltero (degenerado quando trs pontos forem
colineares) ou um tringulo com um ponto de M em seu interior. No caso do quadriltero como
pelo menos um dos quatro ngulos internos 90 basta escolhermos o vrtice com este
ngulo e os adjacentes a ele. No caso do tringulo vamos olhar para o ponto no interior. Esse
ponto olha para os trs lados do tringulo com ngulos que somados resultam em 360 . Pelo
menos um deles 120 . Seja XYZ um tringulo com XYZ 90 . Pela lei dos cossenos
temos: y 2

x2

z2

2 xz cos XYZ

x2

z 2 . Como y 1

x ou z

b. Seja r AB a maior distncia entre dois pontos de M. Tracemos crculos de raio r


centrados em A e B. M dever estar contido em cada um desse crculos. Ento M dever estar
contido na regio de interseo entre eles. Tracemos um crculo de raio

3c
2

3
centrado no
2

ponto mdio C de AB. Veja que este novo crculo cobre a regio de interseo.

c. Vamos usar a mesma idia do problema dos


anis. Se dois ponto de M esto em um crculo de
raio r ento a distncia entre eles no pode ser
maior ou igual a 2r . Ento nosso objetivo
mostrar que existem dois pontos de M dentro de
2
um crculo de raio
(3 n
3)
Seja C um crculo de raio

3
que cobre M. Centrado em cada ponto de M tracemos um crculo
2

de raio r . Suponha que Z um ponto na interseo de dois desses crculos. Ento o crculo de
centro Z e raio r cobre dois pontos de M(os centros dos crculos que cobriam Z). Como mostrar
pelo menos dois desses crculos que traamos iro se intersectar para r

aumentar o raio de C e obter um crculo D de raio

3
2

2
(3 n

3)

? Vamos

r com mesmo centro. Todos esses

crculos estaro contidos em D. Se a rea de D for menor que a soma das reas de cada crculo
com certeza pelo menos dois deles tero interseo. Mas isso acontece se
2

3n

3
2

Como a maior raiz

r 2 (3n 1)

3r

3
4

0 . Agora basta fazermos o estudo do sinal.

3 3 n
e 3n 1 0 se r
2(3n 1)

3 3 n
OK!
2(3n 1)

(Veja que podemos melhorar um pouco a cota do problema, pois 2

3
).
2

3
2(3 n

3)

PROBLEMA 30

(Ibero 97) Seja P

{P1 , P2 ,..., P1997 } um conjunto de 1997 pontos no interior de um crculo de

raio 1, com P1 sendo o centro do crculo. Para k

1,2,...,1997 , seja x k a distncia de Pk ao

ponto de P mais prximo de Pk . Mostre que:

x1

x2

... x1997

9.

SOLUO

Note que x k 1 , para todo k.


Para cada ponto Pk , considere uma
circunferncia k de centro Pk e raio
xk
.
2
Sendo assim, todos essas circunferncias
se tocam em no mximo 1 ponto e esto
no interior de uma circunferncia de
centro P1 e raio 3. Logo,

P1
Pn

[ 1] [

donde:

x1

x2

xn

x12

x 22

x n2

9.

PROBLEMA 31

(IMO 89) So dados n e k inteiros positivos e um conjunto S de n pontos no plano tais que
(i) no h trs pontos em S colineares,
(ii) Para qualquer ponto P de S existem pelo menos k pontos de S equidistantes de P.
Prove que

1
2

2n .

This document was created with Win2PDF available at http://www.win2pdf.com.


The unregistered version of Win2PDF is for evaluation or non-commercial use only.

Вам также может понравиться